Difference between revisions of "2007 AMC 8 Problems/Problem 8"

(Solution)
(3 intermediate revisions by 2 users not shown)
Line 6: Line 6:
 
<math>\triangle BEC</math>.
 
<math>\triangle BEC</math>.
  
<center>[[Image:AMC8_2007_8.png]]</center>
+
<asy>
 +
defaultpen(linewidth(0.7));
 +
pair A=(0,3), B=(3,3), C=(6,0), D=origin, E=(3,0);
 +
draw(E--B--C--D--A--B);
 +
draw(rightanglemark(A, D, C));
 +
label("$A$", A, NW);
 +
label("$B$", B, NW);
 +
label("$C$", C, SE);
 +
label("$D$", D, SW);
 +
label("$E$", E, NW);
 +
label("$3$", A--D, W);
 +
label("$3$", A--B, N);
 +
label("$6$", E, S);</asy>
  
 
<math>\text{(A)}\ 3 \qquad \text{(B)}\ 4.5 \qquad \text{(C)}\ 6 \qquad \text{(D)}\ 9 \qquad \text{(E)}\ 18</math>
 
<math>\text{(A)}\ 3 \qquad \text{(B)}\ 4.5 \qquad \text{(C)}\ 6 \qquad \text{(D)}\ 9 \qquad \text{(E)}\ 18</math>
 
  
 
== Solution ==
 
== Solution ==
Line 19: Line 30:
 
We are trying to find the area of <math>\triangle BEC</math>.
 
We are trying to find the area of <math>\triangle BEC</math>.
  
So, <math>\frac{1}{2} * 3 * 3 = 4.5</math>, <math>\boxed{B}</math>
+
So, <math>\frac{1}{2} \cdot 3 \cdot 3 = \boxed{\textbf{(B)}\ 4.5}</math>
 
 
<center>[[Image:AMC8_2007_8S.png]]</center>
 
  
 
==See Also==
 
==See Also==
 
{{AMC8 box|year=2007|num-b=7|num-a=9}}
 
{{AMC8 box|year=2007|num-b=7|num-a=9}}
 +
{{MAA Notice}}

Revision as of 01:24, 5 July 2013

Problem

In trapezoid $ABCD$, $AD$ is perpendicular to $DC$, $AD$ = $AB$ = $3$, and $DC$ = $6$. In addition, $E$ is on $DC$, and $BE$ is parallel to $AD$. Find the area of $\triangle BEC$.

[asy] defaultpen(linewidth(0.7)); pair A=(0,3), B=(3,3), C=(6,0), D=origin, E=(3,0); draw(E--B--C--D--A--B); draw(rightanglemark(A, D, C)); label("$A$", A, NW); label("$B$", B, NW); label("$C$", C, SE); label("$D$", D, SW); label("$E$", E, NW); label("$3$", A--D, W); label("$3$", A--B, N); label("$6$", E, S);[/asy]

$\text{(A)}\ 3 \qquad \text{(B)}\ 4.5 \qquad \text{(C)}\ 6 \qquad \text{(D)}\ 9 \qquad \text{(E)}\ 18$

Solution

We know that $ABED$ is a square with side length $3$. We subtract $DC$ and $DE$ to get the length of $EC$.

$EC = DC - DE = 6 - 3 = 3$

We are trying to find the area of $\triangle BEC$.

So, $\frac{1}{2} \cdot 3 \cdot 3 = \boxed{\textbf{(B)}\ 4.5}$

See Also

2007 AMC 8 (ProblemsAnswer KeyResources)
Preceded by
Problem 7
Followed by
Problem 9
1 2 3 4 5 6 7 8 9 10 11 12 13 14 15 16 17 18 19 20 21 22 23 24 25
All AJHSME/AMC 8 Problems and Solutions

The problems on this page are copyrighted by the Mathematical Association of America's American Mathematics Competitions. AMC logo.png